Calculus - Hard Volume Problem :\

Click For Summary
The discussion revolves around calculating the volume of water in a rectangular pool with specific dimensions and a depth function. The user formulates a definite integral to find the volume of one section of the pool and then multiplies it by the width to find the total volume. There is some reassurance from other participants that the approach is correct, despite the user's initial doubts about their interpretation of the problem. The integral setup and evaluation are confirmed as appropriate for the given scenario. Overall, the conversation highlights the collaborative effort in solving calculus problems.
calculusisfun
Messages
30
Reaction score
0

Homework Statement


A pool in the shape of a rectangle is ten (10) m wide and twenty five (25) m long. The depth of the pool water x meters from the shallow part/end of the pool is 1 + (x^2)/175 meters.

Write a definite integral that yields the volume of water in the rectangular pool exactly. And then evaluate this integral.

2. The attempt at a solution

So, to find one section's volume I take the following integral: [PLAIN]http://img801.imageshack.us/img801/5991/calc1.png

So, that gives me one of the 25 foot long section's volumes. Thus, I multiply that integral by ten to yield the following: [PLAIN]http://img80.imageshack.us/img80/1236/calc2.png

I'm not sure if I interpreted the question the right way. Any explanations/help would be greatly appreciated. :)
 
Last edited by a moderator:
Physics news on Phys.org
Looks fine to me.
 
Didn't you already ask 2 days ago? It was fine by then, and it remains fine now :smile:
 
Haha okay thanks. Yeah, sorry micromass, I just wanted to get a little more input as I severely doubted I would have gotten it on my first try. :p

Thanks a bunch guys. :)
 
Question: A clock's minute hand has length 4 and its hour hand has length 3. What is the distance between the tips at the moment when it is increasing most rapidly?(Putnam Exam Question) Answer: Making assumption that both the hands moves at constant angular velocities, the answer is ## \sqrt{7} .## But don't you think this assumption is somewhat doubtful and wrong?

Similar threads

Replies
4
Views
4K
  • · Replies 3 ·
Replies
3
Views
6K
  • · Replies 1 ·
Replies
1
Views
3K
  • · Replies 2 ·
Replies
2
Views
5K
  • · Replies 10 ·
Replies
10
Views
3K
  • · Replies 1 ·
Replies
1
Views
3K
  • · Replies 6 ·
Replies
6
Views
4K
  • · Replies 13 ·
Replies
13
Views
3K
  • · Replies 1 ·
Replies
1
Views
3K
  • · Replies 6 ·
Replies
6
Views
3K